Mathcenter Forum  

Go Back   Mathcenter Forum > คณิตศาสตร์โอลิมปิก และอุดมศึกษา > อสมการ
สมัครสมาชิก คู่มือการใช้ รายชื่อสมาชิก ปฏิทิน ข้อความวันนี้

ตั้งหัวข้อใหม่ Reply
 
เครื่องมือของหัวข้อ ค้นหาในหัวข้อนี้
  #1  
Old 01 ตุลาคม 2009, 13:36
LightLucifer's Avatar
LightLucifer LightLucifer ไม่อยู่ในระบบ
กระบี่ธรรมชาติ
 
วันที่สมัครสมาชิก: 25 กันยายน 2008
ข้อความ: 2,352
LightLucifer is on a distinguished road
Default ปัญหาของการพิสูจน์อสมการ 2

คือตอนนี้มกำลังมึนๆกับพวกอสมการ Holder's, Chebychev's, Minkowski's, Power mean อ่ะครับ รบกวนช่วยสอนพวกเทคนิคการพิสูจน์ของอสมการพวกนี้หน่อยนะครับ
ขอข้อนี้ก่อนนะครับ

กำหนดให้ $a,b\geqslant0$ จงพิสูจน์ว่า
$$(a+b)(a^3+b^3)(a^7+b^7)\leqslant 4(a^{11}+b^{11})$$
__________________
เหนือฟ้ายังมีฟ้าแต่เหนือข้าต้องไม่มีใคร

ปีกขี้ผื้งของปลอมงั้นสินะ


...โลกนี้โหดร้ายจริงๆ มันให้ความสุขกับเรา แล้วสุดท้าย มันก็เอาคืนไป...
ตอบพร้อมอ้างอิงข้อความนี้
  #2  
Old 01 ตุลาคม 2009, 21:49
nooonuii nooonuii ไม่อยู่ในระบบ
ผู้พิทักษ์กฎทั่วไป
 
วันที่สมัครสมาชิก: 25 พฤษภาคม 2001
ข้อความ: 6,408
nooonuii is on a distinguished road
Default

Show that $(a+b)(a^3+b^3)\leq 2(a^4+b^4)$

and $(a^4+b^4)(a^7+b^7)\leq 2(a^{11}+b^{11})$

Chebychev inequality.
__________________
site:mathcenter.net คำค้น
ตอบพร้อมอ้างอิงข้อความนี้
  #3  
Old 01 ตุลาคม 2009, 21:52
nooonuii nooonuii ไม่อยู่ในระบบ
ผู้พิทักษ์กฎทั่วไป
 
วันที่สมัครสมาชิก: 25 พฤษภาคม 2001
ข้อความ: 6,408
nooonuii is on a distinguished road
Default

Another Idea : Use Power Mean Inequality.

$a+b\leq ? (a^{11}+b^{11})^{?}$

$a^3+b^3\leq ? (a^{11}+b^{11})^{?}$

$a^7+b^7\leq ? (a^{11}+b^{11})^{?}$
__________________
site:mathcenter.net คำค้น
ตอบพร้อมอ้างอิงข้อความนี้
  #4  
Old 01 ตุลาคม 2009, 22:09
LightLucifer's Avatar
LightLucifer LightLucifer ไม่อยู่ในระบบ
กระบี่ธรรมชาติ
 
วันที่สมัครสมาชิก: 25 กันยายน 2008
ข้อความ: 2,352
LightLucifer is on a distinguished road
Default

อ๋อ ขอบคุณมากเลยครับ
แล้วถ้าเป็นข้อนี้อ่ะครับ เอาแบบไม่ใช้โคชีอ่ะครับคิดไม่ออกอ่ะ

กำหนดให้ $a,b,c\in \mathbb{R}^+$ จงพิสูจน์ว่า
$$a+b+c\leqslant \frac{a^2+b^2}{2c}+\frac{b^2+c^2}{2b}+\frac{c^2+a^2}{2a}$$
__________________
เหนือฟ้ายังมีฟ้าแต่เหนือข้าต้องไม่มีใคร

ปีกขี้ผื้งของปลอมงั้นสินะ


...โลกนี้โหดร้ายจริงๆ มันให้ความสุขกับเรา แล้วสุดท้าย มันก็เอาคืนไป...
ตอบพร้อมอ้างอิงข้อความนี้
  #5  
Old 02 ตุลาคม 2009, 00:00
nooonuii nooonuii ไม่อยู่ในระบบ
ผู้พิทักษ์กฎทั่วไป
 
วันที่สมัครสมาชิก: 25 พฤษภาคม 2001
ข้อความ: 6,408
nooonuii is on a distinguished road
Default

Cauchy-Schwarz
__________________
site:mathcenter.net คำค้น
ตอบพร้อมอ้างอิงข้อความนี้
  #6  
Old 02 ตุลาคม 2009, 00:13
LightLucifer's Avatar
LightLucifer LightLucifer ไม่อยู่ในระบบ
กระบี่ธรรมชาติ
 
วันที่สมัครสมาชิก: 25 กันยายน 2008
ข้อความ: 2,352
LightLucifer is on a distinguished road
Default

ขอแบบที่ไม่ใช่ Cauchy-Schwarz อ่ะครับเพราะอันนี้มันเป็นแบบฝึกหัดของ Chebychev อ่ะครับ
__________________
เหนือฟ้ายังมีฟ้าแต่เหนือข้าต้องไม่มีใคร

ปีกขี้ผื้งของปลอมงั้นสินะ


...โลกนี้โหดร้ายจริงๆ มันให้ความสุขกับเรา แล้วสุดท้าย มันก็เอาคืนไป...
ตอบพร้อมอ้างอิงข้อความนี้
  #7  
Old 02 ตุลาคม 2009, 01:32
nooonuii nooonuii ไม่อยู่ในระบบ
ผู้พิทักษ์กฎทั่วไป
 
วันที่สมัครสมาชิก: 25 พฤษภาคม 2001
ข้อความ: 6,408
nooonuii is on a distinguished road
Default

อ้างอิง:
ข้อความเดิมเขียนโดยคุณ LightLucifer View Post
อ๋อ ขอบคุณมากเลยครับ
แล้วถ้าเป็นข้อนี้อ่ะครับ เอาแบบไม่ใช้โคชีอ่ะครับคิดไม่ออกอ่ะ

กำหนดให้ $a,b,c\in \mathbb{R}^+$ จงพิสูจน์ว่า
$$a+b+c\leqslant \frac{a^2+b^2}{2c}+\frac{b^2+c^2}{2b}+\frac{c^2+a^2}{2a}$$
It should be

$$a+b+c\leqslant \frac{a^2+b^2}{2c}+\frac{b^2+c^2}{2a}+\frac{c^2+a^2}{2b}$$
__________________
site:mathcenter.net คำค้น
ตอบพร้อมอ้างอิงข้อความนี้
  #8  
Old 02 ตุลาคม 2009, 01:38
nooonuii nooonuii ไม่อยู่ในระบบ
ผู้พิทักษ์กฎทั่วไป
 
วันที่สมัครสมาชิก: 25 พฤษภาคม 2001
ข้อความ: 6,408
nooonuii is on a distinguished road
Default

WLOG, assume $a\leq b\leq c$. Then

$\dfrac{1}{c}\leq\dfrac{1}{b}\leq\dfrac{1}{a}$ and $a^2+b^2\leq a^2+c^2\leq b^2+c^2$.

Apply Chebychev inequality to these sets of variables.

Then use AM-HM or other methods to get the required inequality.
__________________
site:mathcenter.net คำค้น
ตอบพร้อมอ้างอิงข้อความนี้
  #9  
Old 02 ตุลาคม 2009, 16:39
LightLucifer's Avatar
LightLucifer LightLucifer ไม่อยู่ในระบบ
กระบี่ธรรมชาติ
 
วันที่สมัครสมาชิก: 25 กันยายน 2008
ข้อความ: 2,352
LightLucifer is on a distinguished road
Default

จากที่พี่ nooonuii แนะมา ผมได้แบบนี้อ่ะครับ
$(\frac{1}{3}((b^2+c^2)+(a^2+c^2)+(a^2+b^2))(\frac{1}{3}(\frac{1}{2a}+\frac{1}{2b}+\frac{1}{2c}) \leqslant \frac{1}{3}(\frac{b^2+c^2}{2a}+\frac{a^2+c^2}{2b} \frac{a^2+b^2}{2c})$
$\Leftrightarrow \frac{1}{3}(a^2+b^2+c^2)(\frac{1}{a}+\frac{1}{b}+\frac{1}{c})\leqslant(\frac{b^2+c^2}{2a}+\frac{a^2+c^2}{2b} \frac{a^2+b^2}{2c}) $

แต่ผมคิดให้ $a+b+c \leqslant LHS$ ไม่ได้อ่ะครับช่วยแนะหน่อยครับ
__________________
เหนือฟ้ายังมีฟ้าแต่เหนือข้าต้องไม่มีใคร

ปีกขี้ผื้งของปลอมงั้นสินะ


...โลกนี้โหดร้ายจริงๆ มันให้ความสุขกับเรา แล้วสุดท้าย มันก็เอาคืนไป...
ตอบพร้อมอ้างอิงข้อความนี้
  #10  
Old 02 ตุลาคม 2009, 22:59
nooonuii nooonuii ไม่อยู่ในระบบ
ผู้พิทักษ์กฎทั่วไป
 
วันที่สมัครสมาชิก: 25 พฤษภาคม 2001
ข้อความ: 6,408
nooonuii is on a distinguished road
Default

Use

$\dfrac{1}{3}(\dfrac{1}{a}+\dfrac{1}{b}+\dfrac{1}{c})\geq \dfrac{3}{a+b+c}$ and

$3(a^2+b^2+c^2)\geq (a+b+c)^2$
__________________
site:mathcenter.net คำค้น
ตอบพร้อมอ้างอิงข้อความนี้
  #11  
Old 02 ตุลาคม 2009, 23:10
LightLucifer's Avatar
LightLucifer LightLucifer ไม่อยู่ในระบบ
กระบี่ธรรมชาติ
 
วันที่สมัครสมาชิก: 25 กันยายน 2008
ข้อความ: 2,352
LightLucifer is on a distinguished road
Default

อ๋อ ผมยังด้อยประสบการณ์อีกมากเลยนะครับเนี่ย ไว้มีสงสัยอีกจะมาถามนะครับ
__________________
เหนือฟ้ายังมีฟ้าแต่เหนือข้าต้องไม่มีใคร

ปีกขี้ผื้งของปลอมงั้นสินะ


...โลกนี้โหดร้ายจริงๆ มันให้ความสุขกับเรา แล้วสุดท้าย มันก็เอาคืนไป...
ตอบพร้อมอ้างอิงข้อความนี้
  #12  
Old 03 ตุลาคม 2009, 10:58
LightLucifer's Avatar
LightLucifer LightLucifer ไม่อยู่ในระบบ
กระบี่ธรรมชาติ
 
วันที่สมัครสมาชิก: 25 กันยายน 2008
ข้อความ: 2,352
LightLucifer is on a distinguished road
Default

ผมขอ Chebychev อีกข้อนะครับ ยังไม่คล่องพอเลย

ให้ $a,b,c\geqslant0$ จงพิสูจน์ว่า
$$(ab+bc+ca)(a+b+c)^4\leqslant27(a^3+b^3+c^3)^2$$
__________________
เหนือฟ้ายังมีฟ้าแต่เหนือข้าต้องไม่มีใคร

ปีกขี้ผื้งของปลอมงั้นสินะ


...โลกนี้โหดร้ายจริงๆ มันให้ความสุขกับเรา แล้วสุดท้าย มันก็เอาคืนไป...

03 ตุลาคม 2009 10:59 : ข้อความนี้ถูกแก้ไขแล้ว 1 ครั้ง, ครั้งล่าสุดโดยคุณ LightLucifer
ตอบพร้อมอ้างอิงข้อความนี้
  #13  
Old 03 ตุลาคม 2009, 11:59
nooonuii nooonuii ไม่อยู่ในระบบ
ผู้พิทักษ์กฎทั่วไป
 
วันที่สมัครสมาชิก: 25 พฤษภาคม 2001
ข้อความ: 6,408
nooonuii is on a distinguished road
Default

ใช้ $ab+bc+ca\leq a^2+b^2+c^2$ ก่อนครับ

จากนั้นก็พิสูจน์สองอสมการคือ

$(a+b+c)(a^2+b^2+c^2)\leq .....$

$(a+b+c)^3\leq .......$
__________________
site:mathcenter.net คำค้น
ตอบพร้อมอ้างอิงข้อความนี้
  #14  
Old 03 ตุลาคม 2009, 13:01
LightLucifer's Avatar
LightLucifer LightLucifer ไม่อยู่ในระบบ
กระบี่ธรรมชาติ
 
วันที่สมัครสมาชิก: 25 กันยายน 2008
ข้อความ: 2,352
LightLucifer is on a distinguished road
Default

ขอบคุณมากเลยครับ ไว้สงสัยจะมาถามใหม่นะครับ ^^
__________________
เหนือฟ้ายังมีฟ้าแต่เหนือข้าต้องไม่มีใคร

ปีกขี้ผื้งของปลอมงั้นสินะ


...โลกนี้โหดร้ายจริงๆ มันให้ความสุขกับเรา แล้วสุดท้าย มันก็เอาคืนไป...
ตอบพร้อมอ้างอิงข้อความนี้
  #15  
Old 03 ตุลาคม 2009, 20:02
Jew's Avatar
Jew Jew ไม่อยู่ในระบบ
บัณฑิตฟ้า
 
วันที่สมัครสมาชิก: 11 กุมภาพันธ์ 2009
ข้อความ: 357
Jew is on a distinguished road
Default

ไหนๆคุณไลท์ก็ตั้งแล้วรบกวนคุณ nooonuii ช่วยคิดหน่อยครับ
จงแสดงว่า
$n[(n+1)^{\frac{1}{n}}-1]\prec 1+\frac{1}{2}+.....+\frac{1}{n}\prec n-[(n-1)(n)^{-\frac{1}{n-1}}]$

เมื่อ n เป็นจำนวนเต็มบวกที่ มากกว่าหรือเท่ากับสอง

ขอแบบภาษาไทยได้ไหมครับ
__________________
สัมหรับคณิตศาสตร์
ผมไม่มีแม้ซึ่งพรสวรรค์ไม่มีแม้โอกาสด้วยอยุ่ต่างจังหวัด
จะมีก็แต่ความรักที่ทุ่มเท....

03 ตุลาคม 2009 20:03 : ข้อความนี้ถูกแก้ไขแล้ว 2 ครั้ง, ครั้งล่าสุดโดยคุณ Jew
ตอบพร้อมอ้างอิงข้อความนี้
ตั้งหัวข้อใหม่ Reply



กฎการส่งข้อความ
คุณ ไม่สามารถ ตั้งหัวข้อใหม่ได้
คุณ ไม่สามารถ ตอบหัวข้อได้
คุณ ไม่สามารถ แนบไฟล์และเอกสารได้
คุณ ไม่สามารถ แก้ไขข้อความของคุณเองได้

vB code is On
Smilies are On
[IMG] code is On
HTML code is Off
ทางลัดสู่ห้อง


เวลาที่แสดงทั้งหมด เป็นเวลาที่ประเทศไทย (GMT +7) ขณะนี้เป็นเวลา 11:04


Powered by vBulletin® Copyright ©2000 - 2024, Jelsoft Enterprises Ltd.
Modified by Jetsada Karnpracha